跳转至

数学分析

极限

上下极限

[[2423Su181045]]

对于数列 \(\{a_n\}_{n\ge1}\) ,定义其上极限\(\lim_{n\rightarrow \infty}\sup_{k\ge n}a_k\)下极限\(\lim_{n\rightarrow \infty}\inf_{k\ge n}a_k\) .

任意数列都存在上下极限,考虑 \(\pm\infty\) .

最直观理解上下极限的方法是如下的等价定义:

\(\{a_n\}_{n\ge1}\) 的一个子列的极限为 \(\{a_n\}_{n\ge1}\) 的一个极限点. 并将 \(\pm\infty\) 也作为极限点. 数列的上(下)极限有等价定义:最大(小)极限点.

上下极限有如下的性质:

\(\{a_n\}_{n\ge1}\) 收敛的充分必要条件为 \(\varliminf_{n\rightarrow \infty}a_n=\varlimsup_{n\rightarrow \infty}a_n\) .

\(\varliminf_{n\rightarrow \infty}-a_n=-\varlimsup_{n\rightarrow \infty}a_n,\ \varlimsup_{n\rightarrow \infty}-a_n=-\varliminf_{n\rightarrow \infty}a_n\) .

\(C>0,\varlimsup_{n\rightarrow \infty}Ca_n=C\varlimsup_{n\rightarrow \infty}a_n\) . \(C<0,\varlimsup_{n\rightarrow \infty}Ca_n=C\varliminf_{n\rightarrow \infty}a_n\) .

上下极限有如下的不等式:

\(\varliminf_{n\rightarrow \infty}a_n\leq \varlimsup_{n\rightarrow \infty}a_n\) .

\(\varlimsup_{n\rightarrow \infty}(x_n+y_n)\leq \varlimsup_{n\rightarrow \infty}x_n+\varlimsup_{n\rightarrow \infty}y_n\) . \(\varliminf_{n\rightarrow \infty}(x_n+y_n)\ge \varliminf_{n\rightarrow \infty}x_n+\varliminf_{n\rightarrow \infty}y_n\) .

证明: \(\sup_{n\ge1}(x_n+y_n)\ge \sup_{n\ge1}x_n+\sup_{n\ge1}y_n\) .

从而可得:

\(\varliminf_{n\rightarrow \infty}(x_n+y_n)\leq \varliminf_{n\rightarrow \infty}x_n+\varlimsup_{n\rightarrow \infty}y_n\leq \varlimsup_{n\rightarrow \infty}(x_n+y_n)\) .

并由上结论的证明过程可以得到:

\(\{x_n\}_{n\geq1},\{y_n\}_{n\ge1}\) ,如果 \(\{y_n\}_{n\ge1}\) 收敛,那么 \(\varliminf_{n\rightarrow \infty}(x_n+y_n)=\varliminf_{n\rightarrow \infty}x_n+\lim_{n\rightarrow \infty}y_n\) , \(\varlimsup_{n\rightarrow \infty}(x_n+y_n)=\varlimsup_{n\rightarrow \infty}x_n+\lim_{n\rightarrow \infty}y_n\) .

连续

柯西:若 \(f\) 为连续函数或者单调函数,并且对于任意 \(x,y\in \mathbb{R}\)\(f(x+y)=f(x)f(y)\) 则存在 \(\alpha\geq0\)\(f(x)=\alpha^x\) .

证明:首先注意到 \(f(x)=f(x/2)^2,\forall x\in \mathbb{R}\) ,所以 \(f\geq0\)\(f(1)=[f(1/n)]^n,\forall n\geq1\) ,设 \(\alpha=f(1)\geq0\) ,从而 \(f(1/n)=\alpha^{1/n}\) ,从而对于任意的有理数 \(\frac{m}{n},m,n\in \mathbb{N}\)\(f\left(\frac{m}{n}\right)=\alpha^{\frac{m}{n}}\) ,并且由 \(f\) 的连续性 / 单调 #imcomplete %%单调怎么推出结论,上界收敛%% 可得 \(f(x)=\alpha^x,\forall x\in \mathbb{R}\) .
^CauchyExp

\(f\) 沿着 \(E\) 连续,如果对于任意的 \(x\in \text{dom}(f)\) ,对任意 \(\epsilon>0\) ,存在 \(\delta>0\) ,对任意 \(y\in E\cap B(x,\delta),\lvert f(x)-f(y)\rvert<\epsilon\) .

\(f\) 分别沿着 \(A,B\) 连续 \(\not\Rightarrow\) \(f\) 沿着 \(A\cup B\) 连续.

考虑 Dirichlet 函数 \(\chi_\mathbb{Q}\) 其沿着 \(\mathbb{Q},\mathbb{R}\backslash \mathbb{Q}\) 连续而在 \(\mathbb{R}\) 上不连续.

\(F\) 是紧集, \(\{f_n\}_{n\geq1}\) 为沿 \(F\) 连续的函数列,若 \(\{f_n\}_{n\geq1}\) 沿着 \(F\) 一致收敛到 \(f\) ,则 \(f\) 沿着 \(F\) 连续.

证明: \(\forall \epsilon>0\) ,存在 \(N\in \mathbb{N},\forall n\geq N(\forall x\in \text{dom}(f) ,\lvert f(x)-f_n(x)\rvert<\frac{\epsilon}{3})\) ,又 \(f_n\) 沿着 \(F\) 连续,所以存在 \(\delta>0,\forall y\in B(x,\delta)\cap F\)\(\lvert f_n(y)-f_n(x)\rvert<\frac{\epsilon}{3}\) ,从而 \(\lvert f(x)-f(y)\rvert\leq \lvert f(x)-f_N(x)\rvert+\lvert f_N(x)-f_N(y)\rvert+\lvert f_N(y)-f(y)\rvert<\epsilon\) .

推论:闭区间上连续函数列一致收敛的极限是连续的.

半连续

check

半连续性(semi-continuity):针对实值函数而言,考虑 \(f:X\rightarrow \bar{\mathbb{R}}=\{-\infty,\infty\}\cup \mathbb{R}\) ,对 \(x_0\in X\) ,如果 \(\forall \epsilon>0\) ,存在开集 \(U\ni x\)\(\forall x\in U\)\(f(x)< f(x_0)+\epsilon\) ,即 \(\varlimsup_{x\rightarrow x_0}f(x)\leq f(x_0)\) ,则称 \(f\)\(x_0\)上半连续;若 \(\forall \epsilon>0\) ,存在开集 \(U\ni x\)\(\forall x\in U\)\(f(x)>f(x_0)-\epsilon\) ,即 \(\varliminf_{x\rightarrow x_0}f(x)\geq f(x_0)\) ,则称 \(f\)\(x_0\)下半连续. 类似可以定义函数在定义域上半连续或者下半连续.

直观上理解,半连续性即为 \(f(x_0)\) 在经过恒正或者恒负的扰动之后,将大于(小于) \(x_0\) 某一邻域内的所有 \(f(x)\) . 或者,半连续性只需简单地理解为只满足极限定义的一半.

半连续性有如下等价条件:

下列命题等价: 1) \(f\) 下半连续; 2) 任给 \(\{x_n\}\rightarrow x:f(x)\leq\liminf_{n\rightarrow \infty}f(x_n)\) ; 3) \(f\)上镜图(epgraph) \(\{(x,y):y\geq f(x)\}\) 是闭集.

证明:
\(1)\Rightarrow 2):\) \(\(\liminf_{n\rightarrow \infty}f(x_n)=\lim_{n\rightarrow \infty}\inf\{f(x_k):k\geq n\}\)\) 又因为 \(f\) 下半连续并且 \(\{x_n\}\) 终在 \(x\) 的任一邻域中,所以 \(\forall \epsilon>0\) ,存在 \(n_\epsilon\in \mathbb{N}\) ,当 \(n>n_\epsilon\) 时,有 \(f(x_n)>f(x)-\epsilon\) ,因此 \(\inf\{f(x_k):k\geq n\}>f(x)-\epsilon\) ,又由 \(\epsilon\) 的任意性可以得到 \(\inf\{f(x_k):k\geq n\}\geq f(x)\) ,从而可以得到结论.

\(2)\Rightarrow 3):\) 考虑该集合中任一收敛序列 \((x_n,y_n):y_n\geq f(x_n)\) ,收敛到 \((x,y)\) ,由 2) 可知 \(f(x)\leq \liminf_{n\rightarrow \infty}f(x_n)\leq y\) ,从而 \((x,y)\in \{(x,y):y\geq f(x)\}\) 为闭集.

\(3)\Rightarrow 1):\) 因为 \(\{(x,y):y\geq f(x)\}\) 是闭集,假设存在 \(x_0\in \text{dom}(f)\) ,存在 \(\epsilon>0\) ,对于任意包含 \(x_0\) 的开集 \(U\) :存在 \(x\in U\) 使得 \(f(x)\leq f(x_0)-\epsilon\) ,则取开集 \(B(x_0,\frac{1}{n})\) 可以得到一个收敛于 \(x_0\) 的序列, \((x_n,f(x_0)-\epsilon)\) 为闭集,并且收敛于 \((x_0,f(x_0)-\epsilon)\) ,所以 \(f(x_0)-\epsilon>f(x_0)\) 显然矛盾.

综上三者等价.

注意到下半连续的定义也可以写为 \(\forall c<f(x_0),\exists\) 开集 \(U\ni x\) 满足 \(\forall x\in U,f(x)>c\) . 因此下半连续还有等价条件:

\(f\)\(x_0\) 下半连续的充分必要条件为 \(\forall c<f(x_0)\) ,集合 \(f^{-1}[(c,+\infty)]=\{x:f(x)>c\}\) 为开集. 进而: \(f\)\(X\) 下半连续的充分必要条件为 \(\forall c\in \mathbb{R}\) ,集合 \(f^{-1}[(c,+\infty)]=\{x:f(x)>c\}\) 为开集.

同理有:

下列命题等价: 1) \(f\) 上半连续; 2) 任给 \(\{x_n\}\rightarrow x:f(x)\geq\limsup_{n\rightarrow \infty}f(x_n)\) ; 3) \(f\)下镜图(hypograph) \(\{(x,y):y\leq f(x)\}\) 是闭集. #imcomplete-whatever 4) \(\forall c\in \mathbb{R}\) ,集合 \(f^{-1}[(-\infty,c)]=\{x:f(x)<c\}\) 为开集.

借助以上等价条件:可以证明半连续性的函数具有如下的运算性质:

| 1) \(f,g\) 下半连续,则 \(f+g\) 下半连续;2) 若 \(f,g\) 下半连续并且 \(f,g>0,\forall x\in \text{dom }f\cap \text{dom }g\) ,则 \(fg\) 下半连续;3) 若 \(f\) 下半连续, \(g\) 连续,则 \(f\circ g\) 下半连续;4) 若 \(\{f_k(x)\}_{k\geq1}\) 下半连续,则 \(g=\sup_{k\geq1}f_k(x)\) 下半连续;若 \(\{f_k(x)\}_{1\leq k\leq n}\) 下半连续,则 \(g=\min_{1\leq k\leq n}f_k(x)\) 下半连续.

证明多数可以借助下半连续函数 \(f\) 诱导的集合 \(\{x:f(x)>c\},\forall c\) 为开集,将问题转化为证明集合相等.
1) \(f,g\) 下半连续,注意到 \(\forall c\in \mathbb{R}:\) \(\(\{x:(f+g)(x)>c\}=\cup_{d\in \mathbb{R}}\{x:(f(x)>c-d)\cap(g(x)>d)\}\)\) 由开集的定义和 \(f,g\) 下半连续的等价条件知 \(\{x:(f(x)>c-d)\cap(g(x)>d)\}\) 为开集. 任意个开集之并仍然为开集. 从而可知 \(f+g\) 下半连续;
2) \(\{x:fg>c\}=\bigcup_{d\in \mathbb{R}^+}\{x:f>d^{-1}c\cap g>d\}\)
3) 对于任意的 \(x\in \{f\circ g(x)>c\}\) ,因为 \(f\) 为下半连续函数,所以 \(\exists\) 开集 \(U\ni g(x)\) ,对于任意的 \(g(x')\in U:g(x')>c\) . 又因为 \(g\) 是连续函数,所以存在 \(U'\ni x\) ,从而 \(\{x:f\circ g>c\}\) 为开集. 证毕.
4) 证明: \(\{x:g>c\}=\cup_{k\geq1}\{x:f_k(x)>c\}\)\(\{x:g>c\}=\cap_{1\leq k\leq n}\{x:f_k(x)>c\}\) .

\(\lVert \cdot\rVert_{l_0}=\#\{i:x_i\neq0\}\) ,则 \(\lVert \cdot\rVert_{l_0}:\mathbb{R}^n\rightarrow \mathbb{R}\) 是下半连续函数.
矩阵的秩函数 \(\text{rank}:\mathbb{R}^{m\times n}\rightarrow \mathbb{R}\) 是下半连续的.

证明: \(c\in \mathbb{R}\) ,注意到 \(\(\{A:\text{rank}(A)>c\}=\bigcup_{r>c,r\in \mathbb{N}}\{A:\text{rank}(A)\geq r\}\)\)

下面证明 \(\{A:\text{rank}(A)\geq r\}\) 为开集,取矩阵范数 \(\lVert A\rVert_{\max}=\max\{\lvert a_{ij}\rvert\}\) ,证明对于任意的 \(A_0\in\{A:\text{rank}(A)\geq r\}\) ,存在 \(\delta>0\) ,对于任意的 \(A'\in\{A:\text{rank}(A)\geq r\}:\lVert A-A'\rVert<\delta\) ,有 \(\text{rank}(A')\geq r\) .

\(\text{rank}(A_0)=0\geq r\) 时,上述条件显然成立;当 \(\text{rank}(A_0)\geq r_0>0\) 时,可知存在 \(A_0\) 的子矩阵,记为 \(A_0^{r_0\times r_0}\) ,满足 \(\det(A_0^{r_0\times r_0})\) ,因为 \(\det\) 是连续函数,所以存在 \(\delta>0\) ,使得 \(\lVert B^{r_0\times r_0}-A^{r_0\times r_0}\rVert<\delta\) 时, \(\det(B^{r_0\times r_0})\neq0\) . 因此 \(\forall C\in \{A:\text{rank}(A)\geq r>0\}\) ,当 \(\lVert C-A\rVert<\delta\) 时, \(\lVert C^{r_0\times r_0}-A^{r_0\times r_0}\rVert<\lVert C-A\rVert<\delta\) ,因此也就可以得到 \(\text{rank}(C)\geq\text{rank}(C)^{r_0\times r_0}\geq r_0\) ,从而得证.

连续函数

间断点

若函数 \(f\)\(x_0\) 点不连续,则称 \(x_0\)\(f\)间断点:如果 \(f(x_0^-),f(x_0^+)\) 均为有限数,而 \(f(x_0^{-})\neq f(x_0^+)\) 或者 \(f(x_0^-)\neq f(x)_)\) 或者 \(f(x_0^+)\neq f(x_0)\) ,则称 \(x_0\)第一类间断点;其他间断点都称为第二类间断点.

单调函数的间断点均为第一类间断点. #imcomplete
区间上单调函数的间断点至多可数.

数列

Stolz定理 其主要解决的是\(\frac{0}{0}\)型和\(\frac{*}{\infty}\)型的极限问题

\(\frac{0}{0}\) )设 \(\{a_n\},\{b_n\}\) 均为无穷小量,其中 \(\{a_n\}\)严格单调减少

微分

微分学基本定理

中值定理
Rolle定理
Largrange中值定理 设\(f\in C([a,b])\),在\((a,b)\)上可微,则存在\(\xi\in(a,b)\),使得:\(\(f'(\xi)=\frac{f(b)-f(a)}{b-a}\)\)

微分学应用

极值理论

定义极大值,极大值点,极小值,极小值点;

极值必要条件:设 \(X_0\)\(f(X)\) 的极值点,如果 \(\frac{\partial{f}}{\partial{x_i}}(X_0)\) 存在,则 \(\frac{\partial{f}}{\partial{x_i}}(X_0)=0\) . 进一步,设 \(f(X)\)\(X_0\) 的一个邻域内的所有二阶偏导数连续,记其在 \(X_0\) 的Hessen 矩阵为 \(H_f(X_0)\) ,则有 1) 如果 \(X_0\)\(f(X)\) 的极小值点,那么 \(H_f(X_0)\geq0\) ,即为半正定对称矩阵;2) 如果 \(X_0\)\(f(X)\) 的极大值点,那么 \(H_f(X_0)\leq 0\) ,即为半负定对称矩阵.

极值充分条件:设 \(f(X)\)\(X_0\) 的一个邻域内的所有二阶偏导数都连续,并且 \(X_0\)\(f(X)\) 的临界点,即 \(\nabla f(X_0)=0\) ,记 \(H_f(X_0)\)\(f(X)\)\(X_0\) 的 Hessen 矩阵,则: 1) \(H_f(X_0)>0\)\(H_f(X_0)\) 为正定矩阵 \(\Rightarrow\) \(X_0\)\(f(X)\) 的极小值点;2) \(H_f(X_0)<0\)\(H_f(X_0)\) 为负定矩阵 \(\Rightarrow\) \(X_0\)\(f(X)\) 的极大值点;3) \(H_f(X_0)\) 为不定矩阵(特征值有正有负) \(\Rightarrow\) \(X_0\) 不是 \(f(X)\) 的极值点;4) \(H_f(X_0)\geq0\)\(H_f(X_0)>0\) 不成立,或者 \(H_f(X_0)\leq0\) 但是 \(H_f(X_0)<0\) 不成立,则需要进一步的信息才能确定 \(X_0\) 是否是 \(f(X)\) 的极值点.

积分

Riemann积分

可积条件

(可积的必要条件) \(f\in R[a,b]\) ,则 \(f\)\([a,b]\) 上有界.

(可积第一充分必要条件)有界函数 \(f\)\([a,b]\) 上可积的充分必要条件为 \(\lim_{\lVert P\rVert\rightarrow 0}\sum\limits_{i=1}^{n}\omega_i\Delta x_i=0\) ,其中 \(P\)\([a,b]\) 上的分划.

第一充分必要条件要求对于 \(\epsilon>0\) ,存在 \(\delta>0\) 任意满足 \(\lVert P\rVert<\delta\) 的分划 \(P\) 都满足 \(\sum\limits_{i=1}^{n}\omega_w\Delta x_i<\epsilon\) .

第二充分必要条件则只要求对于 \(\epsilon>0\) ,存在一个满足条件的分划即可.

(可积第二充分必要条件)有界函数 \(f\)\([a,b]\) 上可积的充分必要条件为 \(\forall \epsilon>0\) ,存在区间 \([a,b]\) 上的一个分划 \(P\) 使得 \(\sum\limits_{P}^{}\omega_i\Delta x_i<\epsilon\) .

此外,可以得到如下可积的充分条件:

\(f\in C[a,b]\Rightarrow f\in R[a,b]\) .

\(f\)\([a,b]\) 上有界并且只有有限个间断点,则 \(f\in R[a,b]\) .

\(f\)\([a,b]\) 上单调,则 \(f\in R[a,b]\) .

广义积分

无穷积分

无穷积分

\(f\)\([a,+\infty)\)上有定义,并且在任何有限区间\([a,u]\)上Riemann可积,如果极限\(\(\lim_{u\rightarrow +\infty}\int_a^uf(x)dx=J\)\)存在,则称该极限\(J\)\(f\)\([a,+\infty)\)上的无穷积分,记为:\(\(J=\int_a^{+\infty}f(x)dx\)\)\(J\)为实数,则称无穷积分\(\int_a^{+\infty}f(x)dx\)收敛,否则称为发散

Newton-Leibniz公式
讨论无穷积分\(\int_1^{+\infty}\frac{dx}{x^p}\)的敛散性.

\(p>1\)收敛,\(p\leq 1\)发散.

换元公式
讨论:\(\int_2^{+\infty}\frac{dx}{x(\ln x)^p}\)的敛散性.

\(\(=\int_{\ln 2}^{+\infty}\frac{dt}{t^p}\)\)转换为上一个例题的问题.

  • (两者的敛散性相同,均为\(p>1\)收敛,\(p\leq 1\)发散)
分部积分公式
讨论:\(\int_{-\infty}^{+\infty}\frac{dx}{1+x^2}\)的敛散性

解答:令\(x=\tan t,t\in(-\frac{\pi}{2},\frac{\pi}{2})\),则有:$$\begin{aligned}

&= \int_{-\infty}0\frac{dx}{1+x2}+\int_0{+\infty}\frac{dx}{1+x2}\
&=\lim_{v\rightarrow \infty}\int_v0\frac{dx}{1+x2}+\lim_{u\rightarrow\infty}\int_0u\frac{dx}{1+x2}\
&=\lim_{v\rightarrow -\infty}(\arctan 0-\arctan v)+\lim_{u\rightarrow +\infty}(\arctan u-\arctan0)\
&=-(-\frac{\pi}{2})+\frac{\pi}{2}=\pi
\end{aligned}$$

  • 注:从中也能看出来积分与断点\(c\)的选取无关. 另外也可以简化为:\(\(\arctan x\bigg |_{-\infty}^{+\infty}\)\)计算.
求极限:\(\lim_{n\rightarrow+\infty}\sin \frac{\pi}{n}\sum\limits_{k=1}^{n}\frac{1}{3+\sin \frac{k \pi}{n}}\)

(用到积分的定义,万能公式)注意到:$$\begin{aligned}

&=\lim_{n\rightarrow+\infty} \frac{\sin\frac{\pi}{n}}{\frac{\pi}{n}}\frac{\pi}{n}\sum\limits_{k=1}^{n}\frac{1}{3+\sin \frac{k \pi}{n}}\
&=\int_0^\pi\frac{dx}{3+\sin x}\
&=\int_0^\pi\frac{dx}{3+\frac{2\tan \frac{x}{2}}{1+\tan^2\frac{x}{2}}}\
&=\int_0^{\pi}\frac{2d(\tan \frac{x}{2})}{3+3\tan^2\frac{x}{2}+2\tan \frac{x}{2}}\
&\overset{u=\tan\frac{x}{2}}{=}\int_0{+\infty}\frac{2du}{3+2u+3u2}\
&=\int_0{+\infty}\frac{2d(u+\frac{1}{3})}{3(u+\frac{1}{3})2+\frac{8}{9}}\
&=\frac{2}{3}\int_{1/3}{+\infty}\frac{dv}{v2+\frac{8}{3}}\
&=\frac{2}{3}\frac{3}{2\sqrt{2}}\arctan(\frac{v}{\frac{2\sqrt{2}}{3}})\bigg|_{1/3}^{+\infty}\
&=\frac{1}{\sqrt{2}}(\frac{\pi}{2}-\arctan\frac{1}{2\sqrt{2}})
\end{aligned}$$

计算:\(\int_0^{+\infty}\frac{dx}{1+x^3}\)

由待定系数法可以得到:$$\begin{aligned}

&=\int_0{+\infty}\left(\frac{1}{3(1+x)}+\frac{(2-x)}{3(x2-x+1)}\right)\
&=\int_0{+\infty}(\frac{1}{3(1+x)}-❗\frac{1}{6}\frac{d(x2-x+1)}{x2-x+1}+\frac{1}{2}\frac{dx}{x2-x+1})\
&=\int_0{+\infty}\left(\frac{1}{3(1+x)}-\frac{1}{6}\frac{d(x2-x+1)}{x2-x+1}+\frac{1}{2}\frac{d(x-\frac{1}{2})}{(x-\frac{1}{2})2+\frac{3}{4}}\right)\
&=0+\frac{1}{2}\frac{2}{\sqrt{3}}\arctan(\frac{2}{\sqrt{3}}(x-\frac{1}{2}))=\frac{2}{9}\sqrt{3}\pi
\end{aligned}$$

证明:\(\int_0^{+\infty}\frac{dx}{1+x^4}=\frac{\pi}{2\sqrt{2}}\)

对于\(\int_0^{+\infty}\frac{dx}{1+x^4}\)的求解见[[积分表]]. 徐森林(N P409)中给出了B函数与\(\Gamma\)函数的公式. Stackexchange

计算:\(I=\int_0^{+\infty}\sin ax e^{-bx}dx,J=\int_0^{+\infty}\cos ax e^{-bx},a>0,b>0\)

[[3-复积分、Cauchy定理]]中给出求解方法,此外也可以直接使用分部积分法解决这一问题. 最终得到:\(\(I=\frac{a}{a^2+b^2},J=\frac{b}{a^2+b^2}\)\)注:这个积分很容易记忆,如果是\(\sin(ax)\)则分子为\(a\),否则为\(b\).

计算Poisson积分\(\int_{-\pi}^\pi\frac{1-r^2}{1-2r\cos x+r^2}dx,0<r<1\)

\(\frac{\pi}{2}\),换元:\(t=\tan\frac{x}{2}\).

证明:Euler-Poisson 积分: \(\int_0^{+\infty}e^{-x^2}dx=\frac{\sqrt{\pi}}{2}\)

(使用Wallis公式和夹逼定理证明)考虑到:$$\begin{aligned}

\frac{(2n)!!}{(2n+1)!!}&=\int_0{\frac{\pi}{2}}\costdt\
&\overset{x=\sin t}{=}\int_01(1-x2)^ndx\
&<\int_01e{-nx2}dx<\int_0e{-nx2}dx\
&<\int_0{+\infty}\frac{dx}{(1+x2)^n}\
&\overset{x=\tan t}{=}\int_0{\frac{\pi}{2}}\cosdt\
&=\frac{(2n-3)
!!}{(2n-2)!!}\frac{\pi}{2}
\end{aligned}\(\(并且:\)\)\int_0{+\infty}e{-nx2}dx\overset{x=\frac{t}{\sqrt{n}}}{=}\frac{1}{\sqrt{n}}\int_0e{-t2}dt\(\(所以有:\)\)n\left(\frac{(2n)!!}{(2n+1)!!}\right)2<\int_0e{-x2}dx<n\left(\frac{(2n-3)!!}{(2n-2)!!}\frac{\pi}{2}\right)^2$$
由Wallis公式\(\(\lim_{n\rightarrow\infty}n\left(\frac{(2n)!!}{(2n+1)!!}\right)^2=\lim_{n\rightarrow \infty}\frac{[(2n)!!]^2}{(2n-1)!!(2n+1)!!}\frac{n}{2n+1}=\frac{\pi}{4}\)\)并且:\(\(\lim_{n\rightarrow \infty}n\frac{[(2n-3)!!]^2}{(2n-2)!!}\cdot\frac{\pi^2}{4}=\lim_{n\rightarrow \infty}\frac{(2n-3)!!(2n-1)!!}{(2n-2)!!}\frac{n}{2n-3}\frac{\pi^2}{4}=\frac{\pi}{4}\)\)最后由夹逼定理即可得到结论.

计算 \(\int_{-\infty}^\infty e^{-\frac{x^2}{2}}dx\) .

解:令 \(I=\int_{-\infty}^\infty e^{-\frac{x^2}{2}}dx\)\(I^2=\int_{-\infty}^{\infty}dx\int_{-\infty}^\infty e^{-\frac{x^2+y^2}{2}}dy\) ,令 \(\left\{\begin{aligned}&x=r\cos \theta \\ &y=r\sin \theta\end{aligned}\right.\) 可得:

\[I^2=\int_{0}^\infty dr\int_{-\pi}^\pi e^{-\frac{r^2}{2}}r d \theta=2\pi\]

从而可得 \(I=\sqrt{2\pi}\) .

无穷积分收敛的Cauchy准则

无穷积分\(\int_a^{+\infty}f(x)dx\)收敛\(\Leftrightarrow\forall \epsilon>0,\exists \Delta>\max\{a,0\}\),当\(u_1,u_2>\Delta\)时有:\(\(\left\lvert \int_a^{u_1}f(x)dx-\int_a^{u_2}f(x)dx \right\rvert=\left\lvert \int_{u_1}^{u_2}f(x)dx\right\rvert<\epsilon\)\)
由函数收敛的Cauchy准则可得,函授收敛的Cauchy准则由数列收敛的Cauchy准则得到.

绝对收敛

\(f\)在任何有限区间\([a,u]\)上Riemann可积,如果\(\int_a^{+\infty}\lvert f(x)\rvert dx\)收敛,则称\(\int_a^{\infty}\)绝对收敛.

绝对值不等式

\(f\)在任何有限区间\([a,u]\)上Riemann可积,并且\(\int_a^{+\infty}\lvert f(x)\rvert dx\)收敛,则\(\int_a^{+\infty}f(x)dx\)也收敛,并且有:\(\(\left\lvert \int_a^{+\infty}f(x)dx\right\rvert\leq \int_a^{+\infty}\lvert f(x)\rvert dx\)\)

  • 注意:前提是\(\int_a^{+\infty}\lvert f(x)\rvert dx\)收敛.
无穷积分的比较判别法

设定义在\([a,+\infty)\)上的两个函数\(f\)\(g\)都在任何有限区间\([a,u]\)上Riemann可积,并且满足:\(\(\lvert f(x)\rvert\leq g(x),\quad x\in[a,+\infty)\)\)则有:1) \(\int_a^{+\infty}g(x)dx\)收敛\(\rightarrow\int_a^{+\infty}\lvert f(x)\rvert dx\)收敛\(\rightarrow \int_a^{+\infty}f(x)dx\)收敛. 2) 当\(\int_a^{+\infty}\lvert f(x)\rvert dx\)发散时\(\rightarrow\)\(\int_a^{+\infty}g(x)dx\)发散.

(无穷积分的比较判别法的极限形式)设\(f,g\)在任何有穷区间\([a,u]\)上Riemann可积,\(g(x)>0\),并且有\(\(\lim_{x\rightarrow +\infty}\frac{\vert f(x)\vert}{g(x)}=c\)\)则有以下三种情况:

1) \(0<c<+\infty\)时,\(\int_{a}^{+\infty}\lvert f(x)\rvert dx\)\(\int_{a}^{+\infty}g(x)dx\)同敛散;
2) \(c=0\)时,\(\int_a^{+\infty}g(x)dx\)收敛\(\rightarrow\int_{a}^{+\infty}\lvert f(x)\rvert dx\)收敛;
3) \(c=+\infty\)时,\(\int_a^{+\infty}g(x)dx\)发散\(\rightarrow\int_a^{+\infty}\lvert f(x)\rvert dx\)发散.

瑕积分

瑕积分

\(f\)在区间\((a,b]\)上有定义,在\(a\)的任一有开邻域内无界,在任何内闭区间\([v,b]\subset(a,b]\)上Riemann可积,如果存在极限:\(\(\lim_{v\rightarrow a^+}\int_v^bf(x)dx=J\)\)则称此极限\(J\)为无界函数\(f\)\((a,b]\)上的瑕积分,记为:\(\(J=\int_a^bf(x)dx\)\)如果\(J\)为实数,则称\(\int_a^bf(x)\)收敛,否则发散,上述点\(a\)称为\(f\)瑕点。类似,可定义瑕点为\(b\)的瑕积分:\(\(\int_a^bf(x)dx=\lim_{u\rightarrow b^{-}}\int_a^bf(x)dx\)\)以及若瑕点为\(c\in(a,b)\)的瑕积分:\(\(\int_a^bf(x)dx=\int_a^cf(x)dx+\int_c^bf(x)dx\)\)以及若\(a,b\)为瑕点,\(f\)在任何内闭区间\([u,v]\subset(a,b)\)上Riemann可积,定义为:\(\(\int_a^bf(x)dx=\int_a^cf(x)dx+\int_c^bf(x)dx\)\)其中\(c\)\((a,b)\)上任何一点👻如何证明与\(c\)的选取无关?|最后,如果有有限多个瑕点,可以将\((a,b)\)分成有限部分,当且仅当每一部分的积分都收敛,\(\int_a^bf(x)dx\)才是收敛的.

Newton-Leibniz公式
换元公式
分部积分公式

含参变量积分

\(\varphi(T)=\int_a^bf(x,T)dx\)其中\(T\)是参变量.

类比函数项级数\(\sum\limits_{n=1}^{\infty}u_n(t)\overset{t_0}{\rightarrow}\sum\limits_{n=1}^{\infty}u_n(t_0)\)数项级数
对参变量\(T\)求极限,求导,求积分;

一致收敛\(f(x,T)\in[a,b]\times D\)\(T_0\)为聚点,\(\forall \epsilon>0,\exists \delta>0\),当\(0<\lvert T-T_0\rvert<\delta\)时,\(\forall x\in[a,b]\),有\(\lvert f(x,T)-h(x)\rvert<\epsilon\),则称为当\(T\rightarrow T_0\)时,\(f(x,T)\)关于\(x\in[a,b]\)一致收敛于\(h(x)\).

保证参变量,自变量构成的二元函数在指定曲序上连续,若存在边界上的瑕点,则补充定义.

含参变量广义积分

\(\int_0^{+\infty}f(x,T)dx=\varphi(T),\forall T\in D\),考虑其一致收敛性.
\(\forall \epsilon>0,\exists M>a\)(与\(T\)无关)使得\(A>M\)时,\(\forall T\in D\)\(\lvert \int_{A}^{+\infty}f(x,T)dx\rvert<\epsilon\),则\(\int_a^{+\infty}f(x,T)dx\)关于\(T\in D\)一致收敛.

\(F(A,T)=\int_a^Af(x,T)dx\),则在\(A|\rightarrow+\infty\)时,\(F(A,T)\)关于\(T\in D\)一致收敛于\(\varphi(T)\),类似可以定义内闭一致收敛.

引入一些特殊函数以表示不定积分、含参变量积分等的解.

含参变量积分求导

考虑变积分限含参变量积分: \(\varphi(t)=\int_{\alpha(t)}^{\beta(t)}f(x,t)dt\) ,其中 \(\alpha,\beta\)\([c,d]\) 上可导,并且 \(a\leq \alpha,\beta\leq b\) ,如果 \(f(x,t),f'_t(x,t)\)\([a,b]\times[c,d]\) 上连续,则 \(\varphi(t)\)\([c,d]\) 上可导,并且 \(\varphi'(t)=\int_{\alpha(t)}^{\beta(t)}f_t'(x,t)dx+f(\beta(t),t)\beta'(t)-f(\alpha(t),t)\alpha'(t)\) .

对于广义含参变量积分,上述结论是否成立? #issue

Gamma 函数

| #imcomplete

\(\Gamma\) 函数: \(\Gamma(s)=\int_0^{+\infty}t^{s-1}e^{-t}dt\) .

\(\Gamma(1)=\int_0^{+\infty}e^{-t}dt=1\) .

注意到 \(\Gamma\) 函数的递推公式: \(\Gamma(s+1)=\int_0^{+\infty}t^se^{-t}dt=s \Gamma(s-1)\) ,特别地 \(\Gamma(n)=(n-1)!\) .

Beta 函数

Beta 函数 / \(B\) 函数:\(B(p,q)=\int_0^1t^{p-1}(1-t)^{q-1}dt\)

\(B(p,q)=\frac{\Gamma(p)\Gamma(q)}{\Gamma(p+q)}\) .
\[B(1/2,1/2)=\int_0^1t^{-\frac{1}{2}}(1-t)^{-\frac{1}{2}} dt\]

级数

数项级数

无穷级数

无穷级数,无穷级数的通项部分和余项,无穷级数收敛,无穷级数发散

  • 数列与无穷级数的关系:数列\(\{a_n\}\)可以得到部分和数列:\(S_n=\sum\limits_{k=1}^{n}a_k\),并且对于\(a_n\),有:\(\(a_n=(a_{n}-a_{n-1})+(a_{n-1}-a_{n-2})+\cdots+(a_2-a_1)+(a_1-a_0)\)\)其中\(a_0=0\),也就是说数列中的每一项可以表示为部分和。从这个观点出发可以利用无穷级数的敛散性解决数列的敛散性问题:
\(1+\frac{1}{2}+\cdots+\frac{1}{n}-\ln n\)的敛散性

\(r_n=1+\frac{1}{2}+\cdots+\frac{1}{n}-\ln n\),则\(a_k=r_k-r_{k-1}=\frac{1}{k}+\ln(1-\frac{1}{k})\).并定义\(r_0=0\),由泰勒公式,\(a_k\sim \frac{1}{k^2}\),所以\(r_n=\sum\limits_{k=1}^{n}a_k\)收敛.

级数收敛的简单性质
  • 收敛数列加减仍然收敛
无穷级数收敛的必要条件
证明\(\sum\limits_{n=1}^{\infty}n\sin \frac{1}{n}\)\(\sum\limits_{n=1}^{\infty}(1+\frac{1}{n})^n\)均发散.

用级数收敛的必要条件.

(无穷级数敛散性的判定)Cauchy定理
级数加上或者去掉有限项仍然收敛
收敛级数的组合级数仍然收敛,并且和相同
(反之)收敛组合级数内的每一个组合都不变号,则原级数收敛,并且和相等

级数\(\((a_1+\cdots+a_{n})+(a_{n_1+1}+\cdots+a_{n_2})+\cdots+(a_{n_{k-1}+1}+\cdots+a_{n_k})+\cdots\)\)在同一个括号内具有相同的符号,则\(\sum\limits_{n=1}^{\infty}a_n\)也收敛,并且两级数和相等.
证明:夹逼定理

发散级数的组合级数不一定收敛
  • \((-1)^n\)
(Cauchy收敛定理)设\(\{a_n\}\)递减并且级数\(\sum\limits_{n=1}^{\infty}a_n\)递减,求证\(\lim_{n\rightarrow+\infty}na_n=0\) NK.14(A).1
判断\(\sum\limits_{n=1}^{\infty}\frac{n^{n+\frac{1}{n}}}{(n+\frac{1}{n})^n}\)的收敛性 NK.14_2.2(16)

issue

\(a_n>0,\{a_n-a_{n+1}\}\)为一个严格减的数列,如果\(\sum\limits_{n=1}^{\infty}a_n\)收敛,证明:\(\lim_{n\rightarrow +\infty}\left(\frac{1}{a_{n+1}}-\frac{1}{a_n}\right)=+\infty\)

证明:(没有巧的方法,只能放缩,同时用到了数列数列与级数的关系)注意到:\(\(\lim_{n \rightarrow\infty}(a_n-a_{n+1})=0\)\)因为\(\{a_n-a_{n+1}\}\)为严格减的数列, #issue 所以 \(a_n-a_{n+1}>0\)\(\(\frac{1}{a_{n+1}}-\frac{1}{a_n}=\frac{a_n-a_{n+1}}{a_na_{n+1}}>\frac{a_n-a_{n+1}}{a_n^2}\)\)❗注意到:\(a_n^2=\sum\limits_{k=n}^{\infty}(a_k^2-a_{k+1}^2)\),这里等式成立的原因是:\(\(a_n^2=\sum\limits_{k=n}^{n+p}(a_k^2-a_{k+1}^2)+a^{2}_{n+p+1}\)\)\(p\rightarrow\infty\)就可以得到上式. 然后有:\(\(a_n^2=\sum\limits_{k=n}^{\infty}(a_k-a_{k+1})(a_k+a_{k+1})<(a_n-a_{n+1})\sum\limits_{k=n}^{\infty}(a_k+a_{k+1})\)\)于是有:\(\(\frac{1}{a_{n+1}}-\frac{1}{a_n}>\frac{1}{\sum\limits_{k=n}^{\infty}(a_k+a_{k+1})}\)\)并由\(\sum\limits_{n=1}^{\infty}a_n\)收敛可以得到\(\sum\limits_{k=n}^{\infty}(a_k+a_{k+1})=R_n+R_{n+1}\),其中\(R_n,R_{n+1}\)\(\sum\limits_{n=1}^{\infty}\)的余项,从而有\(\sum\limits_{k=n}^{\infty}(a_k+a_{k+1})\rightarrow0(n \rightarrow\infty)\),因此可得结论.

研究下面级数的收敛性:\(\sum\limits_{n=1}^{\infty}(\sqrt[n]{1+\frac{1}{n}}-1),\quad \prod_{n=1}^{\infty}\sqrt[n]{1+\frac{1}{n}}\)

参考Stackexchange上的一些解答.
对于后一个无穷乘积注意到:\(\(=1+\left(\sqrt[n]{1+\frac{1}{n}}-1\right),\sqrt[n]{1+\frac{1}{n}}-1\geq 0,n=1,2,\cdots\)\)所以转换为前者的讨论. 有结论:\(\(\sqrt[n]{1+\frac{1}{n}}-1\sim \frac{1}{n^2}\)\)证明方法有很多,1) 如使用不等式:\(\(1\leq 1+\frac{1}{n}\leq (1+\frac{1}{n^2})^n \rightarrow 1\leq \sqrt[n]{1+\frac{1}{n}}\leq (1+\frac{1}{n^2})\)\)从而可得到结论. 2) 另一个角度(同1))则为:\(\(x^n-y^n=(x-y)(x^{n-1}+x^{n-2}y+\cdots+y^{n-1})\)\)或者从 3) 分析的角度,利用中值定理得到不等式 \(e^x-1<ex,x\in[0,1]\)从而可得:\(\(0\leq \sqrt[n]{1+\frac{1}{n}}-1=e^{\frac{1}{n}\log(1+\frac{1}{n})}-1\leq e^{\frac{1}{n^2}}-1\leq \frac{e}{n^2}\)\)或者使用 4) 洛必达法则直接证明同阶$$\begin{aligned}

\lim_{n\rightarrow \infty}\frac{\left(1+\frac{1}{n}\right){\frac{1}{n}}-1}{\frac{1}{n2}}&=\lim_{x \rightarrow0}\frac{(1+x)x-1}{x2}\
&=\lim_{x\rightarrow0}\frac{(1+x)^x \left(\frac{x}{1+x}+\ln(1+x)\right)}{2x}\
&=\lim_{x \rightarrow0}(1+x)^x \left(\frac{1}{2(1+x)}+\frac{\ln(1+x)}{2x}\right)=1
\end{aligned}$$

求级数和:\(\sum\limits_{n=1}^{\infty}\arctan\frac{1}{n^2+n+1}\)

$$\begin{aligned}

=& \sum\limits_{n=1}{\infty}\arctan\frac{(n+1)-n}{n2+n+1}\
=&\sum\limits_{n=1}^{\infty}(\arctan(n+1)-\arctan n)\
=&\lim_{n\rightarrow +\infty}(\arctan(n+1)-\arctan1)\
=&\frac{\pi}{2}-\frac{\pi}{4}=\frac{\pi}{4}
\end{aligned}$$

求解\(\sum\limits_{n=1}^{\infty}nx^{n-1}\)

一种方式是裂项相消,另一种:\(\(\sum\limits_{k=1}^{n}kx^{k-1}=(\sum\limits_{k=1}^{n}x^{k})'=(\frac{x-x^{n+1}}{1-x})'=\frac{nx^{n+1}-(n+1)x^n+1}{(1-x^2)}\)\)然后代入求极限即可.

正项级数

正项级数
(正项级数收敛的充分必要条件,部分和判别法)

正向级数的部分和为有界数列.

(比较判别法)设\(\sum\limits_{n=1}^{\infty}u_n,\sum\limits_{n=1}^{\infty}v_n\)为正项级数,并且存在正整数\(N\)\(n\geq N\)时,\(u_n\leq v_n\),则有以下结论:

1) 若级数\(\sum\limits_{n=1}^{\infty}u_n\)发散,则\(\sum\limits_{n=1}^{\infty}v_n\)发散;
2) 若级数\(\sum\limits_{n=1}^{\infty}v_n\)收敛,则\(\sum\limits_{n=1}^{\infty}u_n\)收敛.

极限形式

\(\sum\limits_{n=1}^{\infty}u_n,\sum\limits_{n=1}^{\infty}v_n\)为正项级数,并且\(\sum\limits_{n=1}^{\infty}v_n\)的每一项均为正数,并且有:\(\(\lim_{n\rightarrow\infty}\frac{u_n}{v_n}=l\)\)其中\(l\geq0\)或者\(l=+\infty\),则:
1) 若\(0<l<\infty\),则级数\(\sum\limits_{n=1}^{\infty}u_n\)与级数\(\sum\limits_{n=1}^{\infty}v_n\)同时收敛或者同时发散;
2) 若\(l=0\),并且级数\(\sum\limits_{n=1}^{\infty}v_n\)收敛,则\(\sum\limits_{n=1}^{\infty}u_n\)也收敛;
3) 若\(l=+\infty\),并且级数\(\sum\limits_{n=1}^{\infty}v_n\)发散,则\(\sum\limits_{n=1}^{\infty}u_n\)也发散。

经常使用的用于比较判别法的级数

$$\begin{aligned}

&\sum\limits_{n=1}{\infty}aqn\
&\sum\limits_{n=1}{\infty}\frac{1}{np}\
&\sum\limits_{n=2}{\infty}\frac{1}{n\lnpn}

\end{aligned}$$

[!note]- 比较判别法的比值形式 设正项级数\(\sum\limits_{}^{}a_n,\sum\limits_{}^{}b_n\),若当\(n\)充分大时其通项满足:\(\(\frac{a_{n+1}}{a_n}\leq \frac{b_{n+1}}{b_n}\)\)则:
1) 若\(\sum\limits_{}^{}b_n\)收敛,则\(\sum\limits_{}^{}a_n\)收敛;
2) 若\(\sum\limits_{}^{}a_n\)发散,则\(\sum\limits_{}^{}b_n\)发散.3

根据比较判别法,有以下两种判定正项级数收敛性的方法,其都是以几何级数\(\sum\limits_{n=1}^{\infty}q^n\)作为比较级数得到的结论.

[!note]- (比较判别法拓展)达朗贝尔(d'Alembert)判别法 设\(D_n=\frac{u_{n+1}}{u_n}\),若存在正整数\(N\),使得\(n>N\)时:

[!note]- 极限形式 \(\lim\limits_{n\rightarrow \infty}\frac{a_{n+1}}{a_n}=d\)
\(d<1\)时级数收敛,当\(d>1\)时级数发散

[!note]- (比较判别法扩展)Cauchy根值判别法 设\(C_n=\sqrt[n]{u_n}\),若存在正整数\(N\),使得\(n>N\)时:1) 存在\(0<q<1,C_n\leq q\) 2) 存在正整数列的子列\(\{n_k\}\),使得\(C_{n_k}\geq 1\),发散

[!note]- 极限形式 \(\varlimsup_{n\rightarrow \infty}\sqrt[n]{a_n}=c\)
\(c<1\)时级数收敛,当\(c>1\)时级数发散.

如果以\(p\)级数作为比较级数,就可以得到下面的方法:

[!note]- (比较判别法扩展)拉阿伯(Raabe)判别法 \(\lim_{n\rightarrow\infty}n\left(\frac{a_n}{a_{n+1}}-1\right)=r\)
\(r>1\)时级数收敛,\(r<1\)时级数发散

如果以\(\sum\limits_{n=1}^{\infty}\frac{1}{n\ln^pn}\)作为比较级数,则可以得到下面判别法:

[!note]- (比较判别法扩展)贝特朗(Bertrand)判别法

[!note]- (比较判别法扩展)Gauss判别法

需要说明的是,上面列举的各种比值类判别法(出现\(\frac{a_n}{a_{n+1}}\)\(\frac{a_{n+1}}{a_n}\))其实都只能应对\(\{a_n\}\)单调的情况(当\(n\)充分大时),而Cauchy根值判别法则可以应对,其适用范围不能被其他的比值判别法覆盖,如下面这一例子:\(\(a_n=2^{-n-(-1)^n}\quad \sum\limits_{n=1}^{\infty}a_n\)\)

判断\(\sum\limits_{n=2}^{\infty}\frac{1}{\ln (n!)}\)的敛散性

解答:由$$\begin{aligned}

\lim_{n\rightarrow \infty}\frac{n\ln n}{\ln (n!)}&=1+\lim_{n\rightarrow\infty}\frac{\ln\frac{n^n}{n!}}{\ln n!}\
&=1+\lim_{n\rightarrow\infty}\frac{n\ln\frac{n}{\sqrt[n]{n!}}}{\ln n!}\
&=1+\lim_{n\rightarrow\infty}\frac{n/e}{\ln n!}\
&=1+\lim_{n\rightarrow\infty}\frac{1/e}{\ln(n+1)}(\text{stolz equation})
\end{aligned}$$

  • 注意:或可以直接\(>\frac{1}{n\ln n}\)
(Raabe)设\(\alpha>0\),证明级数\(\left\lvert \frac{\alpha(\alpha-1)\cdots(\alpha-n+1)}{n!}\right\rvert\)收敛 NK.练习P143.1
对数判别法

待完善

Cauchy积分判别法

\(f(x)\)\([a,+\infty)\)上的非负单调减的连续函数,其中\(a>0\),则级数:\(\(\sum\limits_{n=1}^{\infty}f(a+n)\)\)(或者\(\sum\limits_{n=0}^{\infty}\))与广义积分:\(\(\int_a^{+\infty}f(x)dx\)\)同敛散.
证明:证明不等式:\(\(\sum\limits_{k=0}^{n}f(a+k+1)\leq \sum\limits_{k=0}^{n}\int_{a+k}^{a+k+1}f(x)dx= \int_{a}^{a+n+1}f(x)dx\leq \sum\limits_{k=0}^{n}f(a+k)\)\)

\(\sum\limits_{n=2}^{\infty}\frac{1}{n(\ln n)^p}\)

\(p>1\)时收敛,当\(p\leq1\)时发散;

Riemann函数 \(\zeta(x)=\sum\limits_{n=1}^{\infty}\frac{1}{n^x}\)

\(x>1\)时收敛,在\(x\leq 1\)时发散,则\(\zeta(x)\)定义了\((1,+\infty)\)上的一个函数,称为Riemann函数.

证明:由Cauchy积分比较判别法,\(\zeta(x)\)的敛散性等同于广义积分\(\int_1^{+\infty}\frac{1}{t^x}dt\)的敛散性相同,从而可得到结论.

Cauchy凝聚项判别法 设\(\{a_n\}\)单调减少的正数数列,则\(\sum\limits_{n=1}^{\infty}a_n\)收敛的充分必要条件为\(\sum\limits_{n=0}^{\infty}2^na_{2^n}\)收敛
Sapagof判别法:设正数数列\(\{a_n\}\)单调减少,则\(\lim_{n\rightarrow\infty}a_n=0\)的充分必要条件是正项级数\(\sum\limits_{n=1}^{\infty}\left(1-\frac{a_{n+1}}{a_n}\right)\)发散
(等价形式)设正数数列\(\{a_n\}\)单调增加,则\(\{a_n\}\)与级数\(\sum\limits_{n=1}^{\infty}\left(1-\frac{a_n}{a_{n+1}}\right)\)的敛散性相同
(等价形式)设正项级数\(\sum\limits_{n=1}^{\infty}a_n\)的部分和数列为\(\{S_n\}\),则\(\sum\limits_{n=1}^{\infty}a_n\)\(\sum\limits_{n=1}^{\infty}\frac{a_n}{S_n}\)同敛散
(不存在万能的比较级数)

待完善 Xie.P12

(使用微分中值定理构造裂项不等式,以进行敛散性的判断)
\(\sum\limits_{n=1}^{\infty}a_n\)为正项级数,\(\{S_n\}\)为其部分和数列,\(p>1\),证明:无论\(\sum\limits_{n=1}^{\infty}a_n\)收敛与否,级数\(\sum\limits_{n=1}^{\infty}\frac{a_n}{S_n^p}\)总是收敛的

hint: 对函数\(x^{1-p}\)在区间\([a,b]\)上使用Lagrange中值定理 #未解决

一般项级数

Leibniz判别法

如果\(\{a_n\}\)单调递减并且趋于\(0\),则交错级数\(\sum\limits_{n=1}^{\infty}(-1)^{n-1}a_n\)收敛.

Abel 引理

如果有限数列\(\{a_1,a_2,\cdots,a_m\}\)\(\{b_1,b_2,\cdots,b_m\}\)满足下列条件:1) 数列\(\{a_k\}\)单调;2) \(\lvert B_k\rvert\leq M,k=1,2,\cdots,m\),则有估计式\(\lvert S\rvert=\left\lvert \sum\limits_{k=1}^{m}a_kb_k\right\rvert\leq M(\lvert a_1\rvert+2\lvert a_m\rvert)\)

Abel判别法

\(\sum\limits_{n=1}^{\infty}a_n\)的部分和数列有界,数列\(\{b_n\}\)单调且收敛于\(0\),则\(\sum\limits_{n=1}^{\infty}a_nb_n\)收敛

Dirichlet判别法

\(\sum\limits_{n=1}^{\infty}a_n\)收敛,数列\(\sum\limits_{n=1}^{\infty}b_n\)单调有界,则\(\sum\limits_{n=1}^{\infty}a_nb_n\)收敛

重排级数

定义 \(u_n\) 的正部,负部分别为: \(u_n^+=\frac{\lvert u_n\rvert+u_n}{2},u_n^-=\frac{\lvert u_n\rvert-u_n}{2}\)

设级数 \(\sum\limits_{n=1}^{\infty}a_n\) 条件收敛,求证:\(\lim_{n\rightarrow \infty}\frac{\sum\limits_{k=1}^{n}a_k^+}{\sum\limits_{k=1}^{n}a_k^-}=1\) .
级数绝对收敛和条件收敛与其正部和负部的关系

1) 级数\(\sum\limits_{n=1}^{\infty}u_n\)绝对收敛的充分必要条件为其正部\(\sum\limits_{n=1}^{\infty}u_n^+\)和负部\(\sum\limits_{n=1}^{\infty}u_n^-\)都收敛;
2) 级数\(\sum\limits_{n=1}^{\infty}u_n\)条件收敛的必要条件为其正部和负部都发散

绝对收敛级数交换任意多项或者❗无穷项的次序所得到的新数列仍然绝对收敛,并且两者有相同的和.

首先证明对于收敛的正项级数有此性质,然后利用级数的正部和负部说明.

讨论\(\sum\limits_{n=1}^{\infty}\frac{\sin^2n^{\frac{1}{2023}}}{n}\)的敛散性.

不妨假设原来的级数收敛,因为其为正项级数,所以其重排级数也一定收敛,注意到级数\(\sum\limits_{n=1}^{\infty}\frac{\sin^2n}{n}\)为发散数列,现在对于\(\{\frac{\sin^2n^{\frac{1}{2023}}}{n}\}\)进行重排,使得其前\(n\)项的\(n\)的取值总为:\(1,2^{2023},\cdots,n^{2023}\),对于重排级数,其部分和为\(S_n=\sum\limits_{k=1}^{n}\frac{\sin^2n}{n}\),可知该重排级数发散,矛盾!👻这很违背直觉,有问题吗? #issue

条件收敛级数的重排,Riemann定理

设级数\(\sum\limits_{n=1}^{\infty}a_n\)条件收敛,对于任意实数\(\alpha\)\(\alpha\)可以取\(-\infty,+\infty\)),适当交换其项的次序,可以使得其收敛于\(\alpha\)

绝对收敛级数按照元素乘积之后重排仍然收敛,并且和相同,Cauchy定理

如果级数\(\(\sum\limits_{n=1}^{\infty}a_n,\sum\limits_{n=1}^{\infty}b_n\)\)都绝对收敛,分别等于\(A,B\),则对应两个数列的项的乘积\(a_ib_j(i=1,2,\cdots,j=1,2,\cdots)\)无论按照什么方式排列,得到的级数一定绝对收敛于\(AB\).

证明:证明有上界即可.

  • 实际使用中多采用对角线排列法:\(\(\sum\limits_{n=1}^{\infty}\sum\limits_{j=1}^{n}a_nb_{n+1-j}\quad \sum\limits_{n=0}^{\infty}\sum\limits_{j=1}^{n}a_nb_{n-j}\)\)

无穷乘积

无穷乘积

无穷乘积部分乘积收敛,❗\(\lim_{n\rightarrow \infty}P_n=0,+\infty,-\infty,\infty\)或者不存在则发散.

无穷乘积收敛的必要条件

\(\lim_{n\rightarrow \infty}p_n=1\).

如果\(\sum\limits_{n=1}^{p_n}\)发散于0,不一定有:\(\lim_{n\rightarrow \infty}p_n=1\)

\(p_n=\frac{1}{2}\)\(p_n=\frac{(-1)^n}{2}\)\(p_n=0\).

无穷乘积与无穷级数敛散性的对偶定理

❗设\(p_n>0\),则有:
1) \(\prod_{n=1}^{\infty}p_n\)收敛于\(P>0 \Leftrightarrow \sum\limits_{n=1}^{\infty}\ln p_n\)收敛于\(S=\ln P\).
2) \(\prod_{n-1}^{\infty}p_n\)发散于\(0\Leftrightarrow \sum\limits_{n=1}^{\infty}\ln p_n\)手发散于\(-\infty\).

无穷乘积的等价级数(对于正项或者负项级数)

\(\alpha_n\geq 0\)(或者\(\alpha_n\leq 0,1+\alpha_n>0,n=1,2,\cdots\)),则\(\prod_{n=1}^{\infty}(\alpha_n+1)\)\(\sum\limits_{n=1}^{\infty}\alpha_n\)同敛散.

  • 注意:❗必须保证\(\ln(\alpha_n+1),\alpha_n\)两者同时敛散,因此需要保证其同为正项级数或者负项级数,但可以允许有限项不是.

证明:首先由上面的定理可以得到:\(\prod_{n=1}^{\infty}(\alpha_n+1)\)\(\sum\limits_{n=1}^{\infty}\ln(\alpha_n+1)\)同敛散,并且由同阶关系,可知\(\sum\limits_{n=1}^{\infty}(\alpha_n+1)\)\(\sum\limits_{n=1}^{\infty}\alpha_n\)同敛散.

不满足全正项或者全负项的无穷乘积不满足上述等价级数

\(a_{2n-1}=\frac{1}{\sqrt{n}},a_{2n}=\frac{1}{n}-\frac{1}{\sqrt{n}}\),则无穷乘积\(\prod_{n=1}^{\infty}(1+a_n)\)收敛而\(\sum\limits_{n=1}^{\infty}a_n\)发散. 后者发散易证,前者需要计算偶数部分积然后再做说明.

无穷级数的等价级数(对于变号情形而言)

设级数\(\sum\limits_{n=1}^{\infty}\alpha_n\)收敛,则级数\(\sum\limits_{n=1}^{\infty}\alpha_n^2\)与无穷乘积\(\prod_{n=1}^{\infty}(1+\alpha_n)\)同敛散.

证明:由\(\sum\limits_{n=1}^{\infty}\alpha_n\)收敛,\(\alpha_n\rightarrow0(n \rightarrow 0)\),所以\(\alpha_n-\ln(1+\alpha_n)\sim \frac{1}{2}\alpha_n^2(n \rightarrow0)\).

  • 好处在于,对于一些级数,\(\sum\limits_{n=1}^{\infty}\alpha_n\)收敛能够推出\(\sum\limits_{n=1}^{\infty}\alpha_n^2\)收敛(\(\alpha_n\)\(n\)充分大时不变号就能通过Cauchy收敛原理轻易得到)
\(a_n=\frac{(-1)^{n+1}}{\sqrt{n}}\),求证\(\prod_{n=1}^{\infty}(1+a_n)\)发散

证明:由\(\{a_n\}\)收敛,可知\(\prod_{n=1}^\infty(1+a_n)\)\(\sum\limits_{n=1}^{\infty}a_n^2\)的敛散性相同,从而可得结论.

(无穷乘积“绝对”收敛\(\rightarrow\)无穷乘积收敛)若\(\prod_{n=1}^{\infty}(1+\lvert \alpha_n\rvert)\)收敛,则\(\prod_{n=1}^{\infty}(1+\alpha_n)\)收敛.

证明:由\(\lvert \alpha_n\rvert\geq0\)可以得到:\(\prod_{n=1}^{\infty}(1+\lvert \alpha_n\rvert)\)\(\sum\limits_{n=1}^{\infty}\lvert \alpha_n\rvert\)的敛散性相同,分别由级数绝对收敛的性质和Cauchy收敛定理可以得到\(\sum\limits_{n=1}^{\infty}\alpha_n\)\(\sum\limits_{n=1}^{\infty}\alpha_n^2\)均收敛,从而由上一定理可知\(\prod_{n=1}^{\infty}(1+\alpha_n)\)收敛.

Wallis公式

\(\(\prod_{n=1}^{\infty}\frac{(2n)^2}{(2n-1)(2n+1)}=\frac{1}{2n+1}\left[\frac{(2n)!!}{(2n-1)!!}\right]^2=\frac{\pi}{2}\)\)
进而可以得到下面的结论:
$$\begin{aligned}

&\frac{(2n)!!}{(2n-1)!!}\sim\sqrt{\pi n}\
&\frac{2{2n}(n!)2}{(2n)!}\sim\sqrt{\pi n}
\end{aligned}$$

求解\(\prod_{n=2}^\infty\frac{n^3-1}{n^3+1}\).

解答:$$\begin{aligned}

&=\frac{n-1}{n+1}\cdot\frac{n2+n+1}{n2-n+1}\
&=\frac{n-1}{n+1}\cdot\frac{n(n+1)+1}{n(n-1)+1}\
&=\frac{n+\frac{1}{n+1}}{n+\frac{1}{n-1}}=\frac{1+\frac{1}{n(n+1)}}{1+\frac{1}{(n-1)n}}\
\end{aligned}\(\(从而可以得到无穷乘积等于:\)\)\lim_{n\rightarrow\infty}\frac{1+\frac{1}{(n+1)n}}{1+\frac{1}{2}}=\frac{2}{3}$$

累次级数,每一次的级数和都收敛

函数项级数

定义一致收敛的概念:

称函数列\(\{S_n(x)\}\)在数集 \(E\) 上一致收敛于 \(S(x)\) ,如果:

  • 若函数项级数 \(\sum\limits_{n=1}^{\infty}u_n(x)\) 在开区间 \(I\) 中的每一个有界闭区间上都一致收敛,则称该级数在\(I\)内闭一致收敛
  • 如果\(\sum\limits_{n=1}^{\infty}\lvert u_n(x)\rvert\)在数集\(E\)上一致收敛,则称 \(\sum\limits_{n=1}^{\infty}u_n(x)\)\(E\)绝对一致收敛.
求收敛域

对级数\(\sum\limits_{n=1}^{\infty}u_n(x)\)

  • \(\lim_{n\rightarrow \infty}\sqrt[n]{\lvert u_n(x)\rvert}<1\)收敛,\(>1\)发散
  • \(\lim_{n\rightarrow \infty}\left\lvert \frac{u_{n+1}}{u_n}\right\rvert<1\)收敛,\(>1\)发散
讲义.P41.14.1.1 直观上看一致收敛
函数项级数一致收敛的等价定义

\(f_n\)\(D\)上点点收敛于\(f\),则\(f_n\rightrightarrows f,x\in D\)等价于,对任意\(\{x_n\}\)都有:\(\lim_{n\rightarrow \infty}\lvert f_n(x_n)-f(x)\rvert=0\)

进而可以得到一个证明非一致收敛的方法

\(\epsilon_0,\forall N, n>N,\exists \{x_n\}\in D,\lvert f_n(x_n)-f(x_n)\rvert\geq \epsilon_0\)(只需要找一个子列)

(判断非一致收敛)一致收敛 \(\Rightarrow\) 可换序(反之不成立),逆否:不可以换序\(\Rightarrow\) 不一致收敛
  • \(y=x^n\)\(x\in[0,1)\)上不一致收敛,但是内闭一致收敛.
  • 连续性、求导只需要内闭一致收敛就可以换序(两者都是局部概念)
判定函数项级数一致收敛
Cauchy收敛原理 以及推论:

\(\lVert u_n\rVert\rightarrow0(n\rightarrow \infty),\forall x\in D\)

Weierstrass定理

\(n\)充分大后:\(\(\lvert u_n(x)\rvert\leq M_n,\forall x\in A\)\)\(M_n\)收敛则\(\sum\limits_{}^{}u_n\)一致收敛(绝对一致收敛)

\(\sum\limits_{}^{}\alpha_n \beta_n\)
  • Abel判别法:\(\sum\limits_{n=1}^{\infty}\beta_n\)一致收敛,\(\{\alpha_n\}\)一致有界,\(\forall x\in A\),对于\(n\)单调;
  • Dirichlet判别法:\(\sum\limits_{k=1}^{n}\beta_k\)一致有界,\(\forall x\in A\)\(\alpha_n(x)\rightrightarrows 0\)\(\{\alpha_n\}\)对于\(n\)单调.
Dini定理
Dini定理:对于函数列而言

若在闭区间\([a,b]\)连续函数列\(\{f_n(x)\}\)逐点收敛到连续函数\(f(x)\),并且对于每一个\(x_0\in[a,b]\)\(\{f_n(x_0)\}\)都是单调数列,则\(\{f_n(x)\}\)\([a,b]\)上一致收敛到\(f(x)\)

\(\{u_n(x)\}\)闭区间\([a,b]\)上连续,并且\(\sum\limits_{n=1}^{\infty}u_n(x)\)逐点收敛到\(S(x)\),并且对于每个\(x_0\in[a,b],u_n(x_0)\)不变号,则\(\sum\limits_{n=1}^{\infty}u_n\)一致收敛到\(S(x)\)

一致收敛的函数列的性质
累次极限存在且相等

设函数列\(\{f_n(x)\}\)在区间\((a,b)\)上一致收敛到\(f(x)\),若对于每个\(n\),单侧极限\(\lim_{x\rightarrow a^+}f_n(x)=f(a+0)\)存在,则有:\(\(\lim_{x\rightarrow a^+}\lim_{n\rightarrow \infty}f_n(x)=\lim_{n\rightarrow \infty}\lim_{x\rightarrow a^+}f_n(x)\)\)

推论:逐项连续,则收敛函数连续

设函数列\(\{f_n(x)\}\)在区间\(I\)上一致收敛到\(f(x)\),若每个\(f_n(x)\)在区间\(I\)上连续,则\(f(x)\)\(I\)上连续

积分换序

设函数列\(\{f_n(x)\}\)在闭区间\([a,b]\)上一致收敛到\(f(x)\),并且每一个\(f_n(x)\)都在\([a,b]\)上可积,则\(f(x)\)\([a,b]\)上可积,并且有:\(\(\int_a^bf(x)dx=\lim_{n\rightarrow \infty}\int_a^bf_n(x)dx\)\);并且令\(F_n(x)=\int_a^xf_n(t)dt\),则函数列\(\{F_n(x)\}\)也在\([a,b]\)上一致收敛到\(F(x)=\int_a^xf(t)dt\)

用一致收敛换序的逆否命题. 判定函数项级数:\(\(\sum\limits_{n=1}^{\infty}\frac{nx}{(1+x)(1+2x)\cdots(1+nx)},0\leq x\leq1\)\)的一致收敛性

注意到:\(\(u_n(x)=\frac{1}{(1+x)\cdots(1+(n-1)x)}-\frac{1}{(1+x)\cdots(1+nx)}\)\)从而可以得到级数和为:\(\(\sum\limits_{k=1}^{n}u_k(x)=1-\frac{1}{(1+x)\cdots(1+nx)},x\in[0,1]\)\)\(x=0\)时级数为\(0\),当\(0<x\leq 1\)时级数为\(1\). 注意到该级数在\(0\)点处不连续,而函数\(u_n(x)\)\(0\)点处连续,如果一致收敛则矛盾!所以非一致收敛

判断级数\(\sum\limits_{n=0}^{\infty}x^n\ln x,\sum\limits_{n=0}^{\infty}x^n(\ln x)^2\)\((0,1)\)上是否一致收敛.

1) 注意到:\(\sum\limits_{n=0}^{\infty}x^n\ln x\)\([0,1]\)上内闭一致收敛,由定义:\(\(\sum\limits_{n=0}^{\infty}x^n\ln x=\frac{\ln x}{1-x},x\in(0,1)\)\)显然内闭一致收敛,而当\(x=1\)时级数为\(0\),在\(x=1\)处不连续,而\(x^n\ln x\)\(x=1\)处连续,从而矛盾!
2) 注意到\(u_n'(x)=(x^n(\ln x)^2)'=x^{n-1}\ln x(n\ln x+2)\)\(x=e^{-\frac{2}{n}}\)时取最大,则\(u_n(x)\leq e^{-2}\frac{4}{n^2}\)从而由Weierstrass判别法可以得到级数一致收敛.

NK P140 \(f_n(x)=n^{\alpha}xe^{-n x},n=1,2,\cdots\)\([0,1]\)上1) 收敛;2) 一致收敛;3) 在积分号下取极限可换序\(\lim_{n\rightarrow \infty}\int_0^1f_ndx=\int_0^1 \left\{\lim_{n\rightarrow \infty }f_n\right\}dx\)

1) \(\forall \alpha,f_n\rightarrow0\)
2) \(\(\lVert f\rVert=\sup_{x\in[0,1]}\lvert n^\alpha\cdot xe^{-n x}\rvert=\lvert f_n(\frac{1}{n})\rvert=n^{\alpha-1}e^{-1}\)\)\(\alpha<1\)时一致收敛.
3) 上式右边为\(0\),左边:\(\(\int_0^1f_n(x)dx=\int_0^1n^\alpha xe^{-nx}=n^{\alpha-2}(1-(n+1)e^{-n})\)\)\(\alpha<2\)(注意,一致收敛可换序,但可换序不要求一致收敛)

定义幂级数\(\sum\limits_{n=1}^{\infty}a_n(x-x_0)^n\)是一个函数项级数,可以视为多项式的推广

收敛域:以\(x_0\)为中心的区间,收敛半径:

\[R=\frac{1}{\overline{\lim}_{n\rightarrow \infty}\sqrt[n]{a_n}}=\lim_{n\rightarrow in }\left\lvert \frac{a_n}{a_{n+1}}\right\rvert\]

\(\lvert x-x_0\rvert=R\)时单独讨论(即为对\(\sum\limits_{n=1}^{\infty}a_nR^n\)的收敛性的讨论.)最后得到收敛区间

\(\sum\limits_{n=1}^{\infty}\frac{(-1)^n}{2n-1}\left(\frac{1-x}{1+x}\right)^n\)的收敛域

考虑:\(\(\lim_{n\rightarrow \infty}\frac{1}{\sqrt[n]{2n-1}}\left\lvert \frac{1-x}{1+x}\right\rvert=\left\lvert \frac{1-x}{1+x}\right\rvert<1\)\)时收敛,单独讨论\(x=0\)时,考虑\(\sum\limits_{n=1}^{\infty}\frac{(-1)^n}{2n-1}\)收敛,因此收敛域为\([0,+\infty)\)

\(\sum\limits_{n=1}^{\infty}\frac{x^n}{(1+x)(1+x^2)\cdots(1+x^n)},x\neq-1\)的收敛域

根据达朗贝尔判别法,如果:\(\lim_{n\rightarrow \infty}\left\lvert \frac{a_n}{a_{n+1}}\right\rvert\)存在或者发散到\(+\infty\),则其为幂级数的收敛半径. 该级数的收敛域为\((-\infty,+\infty)-\{-1\}\)

求幂级数\(\sum\limits_{n=1}^{\infty}(1+\frac{1}{2}+\cdots+\frac{1}{n})x^n\)的收敛区间

1) 求半径:\(\(\lim_{n\rightarrow \infty}\sqrt[n]{1+\frac{1}{2}+\cdots+\frac{1}{n}}=\lim_{n\rightarrow \infty}\sqrt[n]{\ln n+r_n}=1\)\)
2) 讨论边界(对数项级数的讨论):\(\sum\limits_{n=1}^{\infty}\frac{1}{n}\)发散,\(-1\)同理;
3) 收敛区间:\((-1,1)\).

\(\sum\limits_{n=1}^{\infty}\frac{2^n\sin^n x}{n^2}\)的收敛域

解:当\(\lvert 2\sin x\rvert\leq 1\)时,显然一致收敛,当\(\lvert 2\sin x\rvert\geq 1\)时不收敛

(幂级数的换序问题)Abel定理

\(\sum\limits_{n=1}^{\infty}a_n(x-x_0)^n\)在收敛区间\(I\)内闭一致收敛,有如下性质:
1) 和函数在\(I\)上连续;
2) \(\forall [a,b]\in I\)逐项可积;
3) 在\((x_0-R,x_0+R)\subseteq I'\subset I\)上逐项可导.

求和:\(\sum\limits_{n=2}^{\infty}\frac{x^n}{n(n-1)}\)

收敛区间为\([-1,1]\),由Abel定理,在\((-1,1)\)上,\(S'(x)=\sum\limits_{n=2}^{\infty}\frac{x^{n-1}}{n-1},S''(x)=\sum\limits_{n=2}^{\infty}x^{n-2}=\frac{1}{1-x}\),从而可以得到:\(\(S'=\int_0^x S''dt-\int_0^x\frac{1}{1-t}dt\)\)

求和:求下列幂级数的和:\(\sum\limits_{n=1}^{\infty}n(n+2)\left(\frac{1}{2}\right)^n\)

解:首先判断收敛半径:\(R=1\),进而容易判定收敛区间为\((-1,1)\). 由Abel定理,可以逐项求导,从而有:\(\(S(x)=\sum\limits_{n=1}^{\infty}(n+1)(n+2)x^n-(n+1)x^n-x^n=\sum\limits_{n=1}^{\infty}(x^{n+1})''-(x^{n+1})'-x^n\)\)由Abel定理:\(\(S(x)=\left(\sum\limits_{n=1}^{\infty}x^{n+2}\right)''-\left(\sum\limits_{n=1}^{\infty}x^{n+1}\right)'-\sum\limits_{n=1}^{\infty}x^n=\left(\frac{x^3}{1-x}\right)''-\left(\frac{x^2}{1-x}\right)'-\frac{x}{1-x}=\frac{3x-x^2}{(1-x)^3}\)\)
这进而可以计算\(\sum\limits_{n=1}^{\infty}n(n+1)\left(\frac{1}{2}\right)^n\)

Fourier级数

幂级数和Fourier级数
  • Taylor级数:任意阶导数,\(x_0\)附近吻合
  • Fourier级数:可积且绝对可积,整体上吻合.
\(f(x)\)的Fourier级数在\([-\pi,\pi]\)上一致收敛于\(f(x)\),求证Parseval等式成立:\(\frac{1}{\pi}\int_{-\pi}^\pi f^2(x)dx=\frac{a_0^2}{2}+\sum\limits_{n=1}^{\infty}(a_n^2+b_n^2)\)
  • Bassel不等式的过程:\(\(\frac{1}{\pi}\int_{-\pi}^\pi\left[f(x)-S_n(x)\right]^2dx=\frac{1}{\pi}\int_{-\pi}^\pi f^2(x)dx-\left[\frac{a_0^2}{2}+\sum\limits_{n=1}^{\infty}(a_n^2+b_n^2)\right]\)\)
  • 或者\(f(x)\)\([-\pi,\pi]\)上连续,从而有界,\(f^2(x)=\frac{a_0}{2}f(x)+\sum\limits_{n=1}^{\infty}[a_nf(x)\cos nx+b_nf(x)\sin nx]\)\([-\pi,\pi]\)上一致收敛,对各项积分可得
求Fourier级数并求和函数的一般步骤
  • 求出级数:$$\begin{aligned}

&a_n=\frac{2}{T}\int_{-\frac{T}{2}}^{\frac{T}{2}}f(x)\cos\frac{2n \pi}{T}xdx\
&b_n=\frac{2}{T}\int_{-\frac{T}{2}}^{\frac{T}{2}}f(x)\sin\frac{2n \pi}{T}xdx
\end{aligned}$$

  • 带入级数:\(\(f(x)\sim \frac{a_0}{2}+\sum\limits_{n=1}^{\infty}(a_n\cos\frac{2n \pi}{T}x+b_n\sin\frac{2n \pi}{T}x)\)\)
  • 依据收敛定理\(\(S(x)=\left\{\begin{aligned} &f(x)\quad x\in(a,b)\text{是连续点}\\ &\frac{f(x+0)+f(x-0)}{2}\quad x\text{是间断点}\\ &\frac{f(a+0)+f(b-0)}{2},\quad x=a,b\\ \end{aligned}\right.\)\)
收敛性的判别方法:
  • Dini

Lipschitz

\(x_0\)左右极限存在,\(\exists L>0,\alpha>0,0<\delta\leq \pi,-\leq u\leq \delta\),使得\(\(\lvert f(x_0\pm u)-f(x_0\pm 0)\rvert\leq Lu^\alpha\)\)\(\bar{f}\)\(x_0\)收敛到\(\(\frac{f(x_0)+0}{f(x_0-0)}\)\)

Dirichlet-Jordan

\(f(x)\)\([x_0-\delta,x_0)\)\((x_0,x_0+\delta]\)上都单调或者可以表示为增函数之差(有界变差),则\(\bar{f}\)\(x_0\)点收敛到:\(\(\frac{f(x_0+0)+f(x_0-0)}{2}\)\)

\(e^x\)展开为Fourier级数

$$\begin{aligned}

&a_n=\frac{1}{\pi}\int_{-\pi}^\pi e^x\cos nxdx=\frac{(-1)n}{n2+1}\frac{e\pi-e\}}{\pi
&b_n=\frac{1}{\pi}\int_{-\pi}^\pi e^x\sin nxdx=\frac{n(-1){n+1}}{n2+1}\frac{e\pi-e}}{\pi
\end{aligned}$$

\([-\pi,\pi]\)上根据Lipschitz判别法:\(\(f(x)\sim\frac{e^\pi-e^{-\pi}}{\pi}\left[\frac{1}{2}+\sum\limits_{n=1}^{\infty}\frac{(-1)^n}{n^2+1}(\cos nx-n\sin nx)\right]=\left\{\begin{aligned} &e^x,x\in(-\pi,\pi)\\ &\frac{e^\pi+e^{-\pi}}{2},x=\pm \pi \end{aligned}\right.\)\)
\(x=\pi\)的:\(\(\frac{e^\pi-e^{-\pi}}{\pi}\left[\frac{1}{2}+\sum\limits_{n=1}^{\infty}\frac{1}{n^2+1}\right]=\frac{e^\pi+e^{-\pi}}{2}\)\)
得到\(\(\sum\limits_{n=1}^{\infty}\frac{1}{n^2+1}=\frac{\pi}{2}\frac{e^\pi+e^{-\pi}}{e^\pi-e^{-\pi}}-\frac{1}{2}\)\)
\(x=0\)得到:\(\(\frac{e^\pi-e^{-\pi}}{\pi}\left(\frac{1}{2}+\sum\limits_{n=1}^{\infty}\frac{(-1)^n}{n^2+1}\right)=1\)\)从而得到:\(\(\sum\limits_{n=1}^{\infty}\frac{(-1)^{n+1}}{n^2+1}=-\left(\frac{\pi}{e^\pi-e^{-\pi}}-\frac{1}{2}\right)\)\)

\(f(x)=\max\{2-\lvert x\rvert,0\},-\pi\leq x<\pi\)

注意到是偶函数,\(b_n=0\)
$$\begin{aligned}

&a_0=\frac{2}{\pi}\int_0^\pi(2-x)dx=\frac{4}{\pi}\
&a_n=\frac{2}{\pi}\int_0^\pi(2-x)\cos nxdx=\frac{2(1-\cos2n)}{n^2\pi},n=1,2,\cdots
\end{aligned}$$

根据Lipschitz判别法:\(\(\hat{f}=\frac{2}{\pi}+\sum\limits_{n=1}^{\infty}\frac{2(1-\cos2n)}{n^2\pi}\cos nx=\max\{2-\lvert x\rvert,0\}\)\)

Fourier级数逐项求积分,逐项求导
  • 逐项求积分:\(f(x)\)\([-\pi,\pi]\)上可积或者绝对可积,\(T=2\pi\)(不要求\(f(x)\)是收敛的),则\(\forall x\in R\),有\(\(\int_0^xf(t)dt=\frac{a_0x}{2}+\sum\limits_{n=1}^{\infty}\int_0^x(a_n\cos nt+b_n\sin nt)dt\)\)
  • 逐项求导:一般不可以逐项求导,除了有限点之外处处可导,可积或者绝对可积
\(f(x)\)\([-\pi,\pi]\)上连续,且其所有Fourier系数均为\(0\),求证\(f(x)\equiv0\)

证明\(\(\int_0^xf(t)dt=\frac{a_0x}{2}+\sum\limits_{n=1}^{\infty}\int_0^x(a_n\cos nt+b_n\sin nt)dt=0\)\)因为\(f(x)\)\([-\pi,\pi]\)上连续,求导即可得到结论.

推论:若\(f,g\)均以\(2\pi\)为周期且连续,且\(f,g\)的Fourier级数相同,则两者相等.
应用举例
  • $$\sum\limits_{n=1}^{\infty}\frac{\sin nx}{n}=\left{\begin{aligned}

&\frac{\pi-x}{2},x\in(0,2\pi)\
&0,x=0,2\pi
\end{aligned}\right.$$

\([0,2\pi]\)上非一致收敛
- \(\(\sum\limits_{n=1}^{\infty}\frac{\cos nx}{n^2}=-\sum\limits_{n=1}^{\infty}\int_0^x\frac{\sin nt}{n}dt=-\int_0^x \sum\limits_{n=1}^{\infty}\frac{\sin nt}{n}dx=\frac{1}{4}x^2-\frac{\pi}{2}x+\frac{\pi^2}{6}\)\)

Riemann引理
利用Riemann引理求极限:$$\begin{aligned}

&1)\lim_{\lambda\rightarrow+\infty}\int_{-\pi}\pi\sin2\lambda xdx\
&2)\lim_{\lambda\rightarrow+\infty}\int_0a\frac{\cos2\lambda x}{1+x}dx
\end{aligned}$$

  • \(\sin^2\lambda x=\frac{1}{2}(1-\cos \lambda x)\)
  • \(\frac{\cos^2\lambda x}{1+x}=\frac{1+\cos2\lambda x}{2(1+x)}\)原极限的值为\(\(\int_0^a\frac{1}{2(1+x)}dx+\lim_{\lambda\rightarrow+\infty}\int_0^a\frac{\cos2\lambda x}{2(1+x)}dx=0\)\)(最后由Reimann引理得到)所以原极限等于:\(\(\frac{1}{2}\ln(1+a)\)\)